The Student Room Group

Will the real TeeEm please stand up!

Scroll to see replies

Original post by TeeEm
it happens ...
well you learned a bit and I have another nice question to add to my books
:smile:


Thank you. :smile:
Reply 1781
Original post by edothero
is SP+TP=2a|SP| + |TP| = 2a a loci property of an ellipse?

Was never taught that :colonhash:


not only you ought to know it but it is examinable !!!
Original post by edothero
is SP+TP=2a|SP| + |TP| = 2a a loci property of an ellipse?

Was never taught that :colonhash:

Though to be fair it is quite obvious..


Yep, it's in the FP3 textbook, went over it a few days ago.
Original post by TeeEm
not only you ought to know it but it is examinable !!!


Thank you anyway.
Reply 1784
well pleasing ... thanks to the people which nominated me and also voted for me
http://www.thestudentroom.co.uk/showthread.php?t=3889039

6 nominations ... two firsts, two seconds and two thirds ....


Thank you again

c17.jpg
Original post by TeeEm
well pleasing ... thanks to the people which nominated me and also voted for me
http://www.thestudentroom.co.uk/showthread.php?t=3889039

6 nominations ... two firsts, two seconds and two thirds ....


Thank you again

c17.jpg


Calm tf down, its only a forum site
Reply 1786
Original post by GeologyMaths
Calm tf down, its only a forum site


you are clearly very jealous ...
a6.jpg
Someone help? Found this question on Variable Acceleration in 2 dimensions and don't understand how they get values of maximum and minimum acceleration for part b. I managed to get a=12 using the given rearrangement, but don't know how to get the maximum!

Posted from TSR Mobile
Reply 1788
Original post by Hassan55
Someone help? Found this question on Variable Acceleration in 2 dimensions and don't understand how they get values of maximum and minimum acceleration for part b. I managed to get a=12 using the given rearrangement, but don't know how to get the maximum!

Posted from TSR Mobile


@Zacken @SeanFM@16Characters....


do you mind please
(edited 8 years ago)
Original post by Hassan55
Someone help? Found this question on Variable Acceleration in 2 dimensions and don't understand how they get values of maximum and minimum acceleration for part b. I managed to get a=12 using the given rearrangement, but don't know how to get the maximum!

Posted from TSR Mobile


Let the acceleration be aa, then acceleration will achieve minimum and maximum values when dadt=v=0\frac{da}{dt} = v = 0, so find the values of tt for which this is so, investigate whether they are minimum or maximum values by considering the second derivative and then plug it back into the equation to find the min/max acceleration.

Edit: ignore this, as per SeanFM, recall that cos2x\cos^2 x varies between 00 and 11...
(edited 8 years ago)
Reply 1791
Original post by Zacken
Aye


thank you
Original post by Hassan55
Someone help? Found this question on Variable Acceleration in 2 dimensions and don't understand how they get values of maximum and minimum acceleration for part b. I managed to get a=12 using the given rearrangement, but don't know how to get the maximum!

Posted from TSR Mobile


Look at the expression and think the smallest and largest value that it can take.
Original post by Zacken
Let the acceleration be aa, then acceleration will achieve minimum and maximum values when dadt=v=0\frac{da}{dt} = v = 0, so find the values of tt for which this is so, investigate whether they are minimum or maximum values by considering the second derivative and then plug it back into the equation to find the min/max acceleration.

Edit: ignore this, as per SeanFM, recall that cos2x\cos^2 x varies between 00 and 11...

Why do you put v=0,can't you put a=0 and instead of using the suggested expression, can't you use dv/da=a=0 and solve for t in both i and j. Is there any other approach,ie with the expression they give, to solve this?
Original post by Hassan55
Why do you put v=0,can't you put a=0 and instead of using the suggested expression, can't you use dv/da=a=0 and solve for t in both i and j. Is there any other approach,ie with the expression they give, to solve this?


I don't know what I was thinking, but da/dt is not v... , themethod you propose would find the maximum/minimum velocity, not acceleration. You could differentitate the position vector they've given you *four* times and set thag equal to 0, but it's a lot of work when you can just look at how cos x varies from the dervived expression!
My proposed method seemed to have worked surprisingly, where I set the i and j components of a to 0 and worked out values for t to get 12 and 14. I used the method similar to this mark scheme.. you're right saying a=0 will give max/min velocity, but how come they do this?

Posted from TSR Mobile
Also I differentiate 4 times (only had to do it twice more because of part a) and got tan(2t)=-4/3... And by getting t I didn't get 12 or 14 :/

Posted from TSR Mobile
Original post by Hassan55
Also I differentiate 4 times (only had to do it twice more because of part a) and got tan(2t)=-4/3... And by getting t I didn't get 12 or 14 :/

Posted from TSR Mobile


Oops, did I say 4 times? My mistake, it was really late at night and I was in bed. Differentitate once to get velocity, second time to get acceleration, third time to get jerk and then that =0. So differentiate thrice, not four times.
Do you differentiate a naturally as you keep differentiating from r or do you differentiate the expression the give you?

Posted from TSR Mobile
I got 32sin(2t)i-24cos(2t)j=0 after differentiating 3 times. Would I then 32sin(2t)=0 and -24cos(2t)j=0 to get values of t in range 0<t<2Pi and sub in to given expression of a to obtain max/min?

Posted from TSR Mobile

Quick Reply

Latest

Trending

Trending